返回列表 发帖

gwd23-39

Q39:

 

In Gandania, where the government has a monopoly on tobacco sales, the incidence of smoking-related health problems has risen steadily for the last twenty years.  The health secretary recently proposed a series of laws aimed at curtailing tobacco use in Gandania.  Profits from tobacco sales, however, account for ten percent of Gandania’s annual revenues.  Therefore, Gandania cannot afford to institute the proposed laws.

 


 


Which of the following, if true, most seriously weakens the argument?

 


 


A  All health care in Gandania is government-funded.

B  Implementing the proposed laws is not likely to cause a significant increase in the amount of tobacco Gandania exports.

C  The percentage of revenue Gandania receives from tobacco sales has remained steady in recent years.

D   Profits from tobacco sales far surpass any other single source of revenue for the Gandanian government.

E  No government official in Gandania has ever previously proposed laws aimed at curtailing tobacco use.

  


为什么是选C呢  如果是不变的 不是说明还是占据了很大的比重么?

收藏 分享

是的是的  我也选a的~~

TOP

 我认为C答案应该是错的,正确答案应该是A,比较好理解:

控制烟草后虽然政府收入减少,但用于烟草带来的上的支出也减少,总体来看政府不一定 can't afford the proposed laws
在网上搜了一下其他论坛上的讨论也基本支持这一看法

TOP

返回列表

站长推荐 关闭


美国top10 MBA VIP申请服务

自2003年开始提供 MBA 申请服务以来,保持着90% 以上的成功率,其中Top10 MBA服务成功率更是高达95%


查看